Use non-breaking space (~) between 'Figure' and the figure number.
[course.git] / latex / problems / Serway_and_Jewett_8 / problem05.25.tex
index 259026cbcbc5859edad4d73e5e1a0095083944ec..9c76b3aba1e1a227d2b05b883df21f3c5e3f8917 100644 (file)
@@ -1,6 +1,6 @@
 \begin{problem*}{5.25}
 A bag of cement whose weight is $F_g$ hangs in equilibrium from three
-wires shown in Figure P5.24.  Two of the wires make angles
+wires shown in Figure~P5.24.  Two of the wires make angles
 $\theta_1=60.0\dg$ and $\theta_2=40.0\dg$ with the horizontal.
 Assuming the system is in equilibrium, show that the tension in the
 left-hand wire is